Limit of $s_n = intlimits_0^1 frac{nx^{n-1}}{1+x} dx$ as $n to infty$












8












$begingroup$


Let $s_n$ be a sequence defined as given below for $n geq 1$. Then find out $limlimits_{n to
infty} s_n$.
begin{align}
s_n = intlimits_0^1 frac{nx^{n-1}}{1+x} dx
end{align}



I have written a solution of my own, but I would like to know it is completely correct, and also I would like if people post more alternative solutions.










share|cite|improve this question











$endgroup$












  • $begingroup$
    Is the downvote because of self-answering?
    $endgroup$
    – user14082
    Feb 1 '13 at 18:08










  • $begingroup$
    Probably. Why did you post a question to which you already knew the answer?
    $endgroup$
    – Todd Wilcox
    Feb 1 '13 at 18:09






  • 2




    $begingroup$
    A few reasons 1. It gives me oppurtunity to verify that the solution is indeed correct. (I self study, so even though I get an answer and I am pretty sure about it, there is no real way to verify the solution completely.) 2. It allows probably other people to offer me better solutions. 3. I can do so on a blog, but then it might not get the same attention on the blog. 4. MSE's editing capabilities are better than almost all other blogging software I have found.
    $endgroup$
    – user14082
    Feb 1 '13 at 18:10








  • 1




    $begingroup$
    I didn't downvote either, but it seems stylistically and logically wrong: the question is "is the following I reproduce in the question text correct?", the answer "yes/no" with reason. I jabs answered my own questions, when I didn't get answers or, after my thought, believed I could improve on other posting; answering your own questions isn't wrong per se. It's also simply more convenient to have the actual question on top.
    $endgroup$
    – gnometorule
    Feb 1 '13 at 18:42






  • 2




    $begingroup$
    +1: I see no problem posting a question for which you already know the answer and asking for alternative approaches.
    $endgroup$
    – Mike Spivey
    Feb 1 '13 at 21:02
















8












$begingroup$


Let $s_n$ be a sequence defined as given below for $n geq 1$. Then find out $limlimits_{n to
infty} s_n$.
begin{align}
s_n = intlimits_0^1 frac{nx^{n-1}}{1+x} dx
end{align}



I have written a solution of my own, but I would like to know it is completely correct, and also I would like if people post more alternative solutions.










share|cite|improve this question











$endgroup$












  • $begingroup$
    Is the downvote because of self-answering?
    $endgroup$
    – user14082
    Feb 1 '13 at 18:08










  • $begingroup$
    Probably. Why did you post a question to which you already knew the answer?
    $endgroup$
    – Todd Wilcox
    Feb 1 '13 at 18:09






  • 2




    $begingroup$
    A few reasons 1. It gives me oppurtunity to verify that the solution is indeed correct. (I self study, so even though I get an answer and I am pretty sure about it, there is no real way to verify the solution completely.) 2. It allows probably other people to offer me better solutions. 3. I can do so on a blog, but then it might not get the same attention on the blog. 4. MSE's editing capabilities are better than almost all other blogging software I have found.
    $endgroup$
    – user14082
    Feb 1 '13 at 18:10








  • 1




    $begingroup$
    I didn't downvote either, but it seems stylistically and logically wrong: the question is "is the following I reproduce in the question text correct?", the answer "yes/no" with reason. I jabs answered my own questions, when I didn't get answers or, after my thought, believed I could improve on other posting; answering your own questions isn't wrong per se. It's also simply more convenient to have the actual question on top.
    $endgroup$
    – gnometorule
    Feb 1 '13 at 18:42






  • 2




    $begingroup$
    +1: I see no problem posting a question for which you already know the answer and asking for alternative approaches.
    $endgroup$
    – Mike Spivey
    Feb 1 '13 at 21:02














8












8








8


5



$begingroup$


Let $s_n$ be a sequence defined as given below for $n geq 1$. Then find out $limlimits_{n to
infty} s_n$.
begin{align}
s_n = intlimits_0^1 frac{nx^{n-1}}{1+x} dx
end{align}



I have written a solution of my own, but I would like to know it is completely correct, and also I would like if people post more alternative solutions.










share|cite|improve this question











$endgroup$




Let $s_n$ be a sequence defined as given below for $n geq 1$. Then find out $limlimits_{n to
infty} s_n$.
begin{align}
s_n = intlimits_0^1 frac{nx^{n-1}}{1+x} dx
end{align}



I have written a solution of my own, but I would like to know it is completely correct, and also I would like if people post more alternative solutions.







sequences-and-series integration limits






share|cite|improve this question















share|cite|improve this question













share|cite|improve this question




share|cite|improve this question








edited Feb 1 '13 at 18:34

























asked Feb 1 '13 at 18:06







user14082



















  • $begingroup$
    Is the downvote because of self-answering?
    $endgroup$
    – user14082
    Feb 1 '13 at 18:08










  • $begingroup$
    Probably. Why did you post a question to which you already knew the answer?
    $endgroup$
    – Todd Wilcox
    Feb 1 '13 at 18:09






  • 2




    $begingroup$
    A few reasons 1. It gives me oppurtunity to verify that the solution is indeed correct. (I self study, so even though I get an answer and I am pretty sure about it, there is no real way to verify the solution completely.) 2. It allows probably other people to offer me better solutions. 3. I can do so on a blog, but then it might not get the same attention on the blog. 4. MSE's editing capabilities are better than almost all other blogging software I have found.
    $endgroup$
    – user14082
    Feb 1 '13 at 18:10








  • 1




    $begingroup$
    I didn't downvote either, but it seems stylistically and logically wrong: the question is "is the following I reproduce in the question text correct?", the answer "yes/no" with reason. I jabs answered my own questions, when I didn't get answers or, after my thought, believed I could improve on other posting; answering your own questions isn't wrong per se. It's also simply more convenient to have the actual question on top.
    $endgroup$
    – gnometorule
    Feb 1 '13 at 18:42






  • 2




    $begingroup$
    +1: I see no problem posting a question for which you already know the answer and asking for alternative approaches.
    $endgroup$
    – Mike Spivey
    Feb 1 '13 at 21:02


















  • $begingroup$
    Is the downvote because of self-answering?
    $endgroup$
    – user14082
    Feb 1 '13 at 18:08










  • $begingroup$
    Probably. Why did you post a question to which you already knew the answer?
    $endgroup$
    – Todd Wilcox
    Feb 1 '13 at 18:09






  • 2




    $begingroup$
    A few reasons 1. It gives me oppurtunity to verify that the solution is indeed correct. (I self study, so even though I get an answer and I am pretty sure about it, there is no real way to verify the solution completely.) 2. It allows probably other people to offer me better solutions. 3. I can do so on a blog, but then it might not get the same attention on the blog. 4. MSE's editing capabilities are better than almost all other blogging software I have found.
    $endgroup$
    – user14082
    Feb 1 '13 at 18:10








  • 1




    $begingroup$
    I didn't downvote either, but it seems stylistically and logically wrong: the question is "is the following I reproduce in the question text correct?", the answer "yes/no" with reason. I jabs answered my own questions, when I didn't get answers or, after my thought, believed I could improve on other posting; answering your own questions isn't wrong per se. It's also simply more convenient to have the actual question on top.
    $endgroup$
    – gnometorule
    Feb 1 '13 at 18:42






  • 2




    $begingroup$
    +1: I see no problem posting a question for which you already know the answer and asking for alternative approaches.
    $endgroup$
    – Mike Spivey
    Feb 1 '13 at 21:02
















$begingroup$
Is the downvote because of self-answering?
$endgroup$
– user14082
Feb 1 '13 at 18:08




$begingroup$
Is the downvote because of self-answering?
$endgroup$
– user14082
Feb 1 '13 at 18:08












$begingroup$
Probably. Why did you post a question to which you already knew the answer?
$endgroup$
– Todd Wilcox
Feb 1 '13 at 18:09




$begingroup$
Probably. Why did you post a question to which you already knew the answer?
$endgroup$
– Todd Wilcox
Feb 1 '13 at 18:09




2




2




$begingroup$
A few reasons 1. It gives me oppurtunity to verify that the solution is indeed correct. (I self study, so even though I get an answer and I am pretty sure about it, there is no real way to verify the solution completely.) 2. It allows probably other people to offer me better solutions. 3. I can do so on a blog, but then it might not get the same attention on the blog. 4. MSE's editing capabilities are better than almost all other blogging software I have found.
$endgroup$
– user14082
Feb 1 '13 at 18:10






$begingroup$
A few reasons 1. It gives me oppurtunity to verify that the solution is indeed correct. (I self study, so even though I get an answer and I am pretty sure about it, there is no real way to verify the solution completely.) 2. It allows probably other people to offer me better solutions. 3. I can do so on a blog, but then it might not get the same attention on the blog. 4. MSE's editing capabilities are better than almost all other blogging software I have found.
$endgroup$
– user14082
Feb 1 '13 at 18:10






1




1




$begingroup$
I didn't downvote either, but it seems stylistically and logically wrong: the question is "is the following I reproduce in the question text correct?", the answer "yes/no" with reason. I jabs answered my own questions, when I didn't get answers or, after my thought, believed I could improve on other posting; answering your own questions isn't wrong per se. It's also simply more convenient to have the actual question on top.
$endgroup$
– gnometorule
Feb 1 '13 at 18:42




$begingroup$
I didn't downvote either, but it seems stylistically and logically wrong: the question is "is the following I reproduce in the question text correct?", the answer "yes/no" with reason. I jabs answered my own questions, when I didn't get answers or, after my thought, believed I could improve on other posting; answering your own questions isn't wrong per se. It's also simply more convenient to have the actual question on top.
$endgroup$
– gnometorule
Feb 1 '13 at 18:42




2




2




$begingroup$
+1: I see no problem posting a question for which you already know the answer and asking for alternative approaches.
$endgroup$
– Mike Spivey
Feb 1 '13 at 21:02




$begingroup$
+1: I see no problem posting a question for which you already know the answer and asking for alternative approaches.
$endgroup$
– Mike Spivey
Feb 1 '13 at 21:02










5 Answers
5






active

oldest

votes


















12












$begingroup$

We simplify the formulate for $s_n$ by integrating by parts.



begin{align}
s_n &= intlimits_0^1 frac{nx^{n-1}}{1+x} d x \
&= left[
frac{1}{1+x} int nx^{n-1} d x
- int frac{1}{left(1+xright)^2} left(int nx^{n-1} d xright) d x
right]^1_0 \
&= left[frac{1}{1+x} int nx^{n-1} d xright]^1_0
- left[int frac{1}{left(1+xright)^2} left(int nx^{n-1} dxright) d xright]^1_0 \
&= left[frac{x^n}{1+x}right]^1_0
- left[int frac{x^n}{left(1+xright)^2} d xright]^1_0 \
&= frac{1}{2} - intlimits_0^1 frac{x^n}{left(1+xright)^2} d x \
end{align}



Now we estimate the remaining integral in the expression
begin{align}
I(n) &= intlimits_0^1 frac{x^n}{left(1+x right)^2} d x \
&leq intlimits_0^1 x^n d x \
&= frac{1}{n+1}
end{align}



Hence, $I(n) to 0$ as $n to infty$.



And so, the expression can be rewritten as
begin{align}
limlimits_{n to infty} s_n = frac{1}{2}
end{align}






share|cite|improve this answer











$endgroup$













  • $begingroup$
    What do you mean by $I(n) = e^{log(x)n}$ in the last line?
    $endgroup$
    – Antonio Vargas
    Feb 1 '13 at 18:15










  • $begingroup$
    Added the explanation. I hope its correct.
    $endgroup$
    – user14082
    Feb 1 '13 at 18:23










  • $begingroup$
    It is not; you can't just pull a $log x$ out of the integral since you're integrating with respect to $x$.
    $endgroup$
    – Antonio Vargas
    Feb 1 '13 at 18:29










  • $begingroup$
    Arrrrgh. Thanks.
    $endgroup$
    – user14082
    Feb 1 '13 at 18:30






  • 4




    $begingroup$
    $I(n)leq int_0^1 x^n dx={1over n+1}.$
    $endgroup$
    – user940
    Feb 1 '13 at 19:19





















9












$begingroup$

We use a basic result in calculus, namely $lim_{nto infty}nint_0^1x^nf(x) dx=f(1)$, $f$ continuous on $[0,1]$
$$lim_{nto infty}left(frac{n}{n-1}times (n-1)intlimits_0^1 x^{n-1} frac{1}{(1+x)} dxright)=frac{1}{2}$$



Chris.






share|cite|improve this answer











$endgroup$













  • $begingroup$
    What is the name or proof of this basic result?
    $endgroup$
    – Alex
    Feb 1 '13 at 19:53






  • 2




    $begingroup$
    @Alex: I don't know if it has a name. Our teacher told us once that the result may be easily proved with the calculus knowledge in high school. This link will help you: math.stackexchange.com/questions/168163/…
    $endgroup$
    – user 1357113
    Feb 1 '13 at 20:01






  • 1




    $begingroup$
    Nice solution Chris'ssister! :-) Your question and the corresponding answers provide a decent tool. :-)
    $endgroup$
    – user14082
    Feb 1 '13 at 20:32












  • $begingroup$
    @OrangeHarvester: Thanks! :-) are you Jayesh? (sister here now)
    $endgroup$
    – user 1357113
    Feb 1 '13 at 20:44












  • $begingroup$
    Yes. I am Jayesh. Name changed for a month.
    $endgroup$
    – user14082
    Feb 1 '13 at 22:15



















5












$begingroup$

Here's a solution based on order statistics, similar to my answer here.



Let $X_1,dots, X_n$ be i.i.d. uniform(0,1) random variables.
The distribution function of $X$ is $F(x)=x$ and density $f(x)=1$ for $0leq xleq 1$.
Now let $M=max(X_1,dots, X_n)$; its density function is
$$f_M(x)=n F(x)^{n-1}f_X(x)=n,x^{n-1}text{ for }0leq xleq 1.$$
Also, it is not hard to see that $Mto 1$ in distribution as $ntoinfty$.
Now $$int_0^1 {n x^{n-1}over 1+x} ,dx =int_0^1 {1over 1+x}, f_M(x) ,dx
=mathbb{E}left({1over 1+M}right).$$



This converges to
${1over 1+1}={1over 2}$ as $ntoinfty$.






share|cite|improve this answer











$endgroup$













  • $begingroup$
    Thanks. A different solution. :-)
    $endgroup$
    – user14082
    Feb 1 '13 at 18:37












  • $begingroup$
    @MikeSpivey Yeah, it's the way I think. Some of the other answers here are great, too.
    $endgroup$
    – user940
    Feb 1 '13 at 21:06










  • $begingroup$
    @Byron: If I may say so, I often enjoy your answers that use probability and statistics to answer questions that appear to be outside of those fields. They are close enough to the way I think that I can appreciate them, but I often wouldn't think of those arguments myself.
    $endgroup$
    – Mike Spivey
    Feb 1 '13 at 21:09










  • $begingroup$
    @MikeSpivey Thanks for the kind words. I learn a lot from your answers, too.
    $endgroup$
    – user940
    Feb 1 '13 at 21:12



















5












$begingroup$

Notice

(1) $frac{s_n}{n} + frac{s_{n+1}}{n+1} = int_0^1 x^{n-1} dx = frac{1}{n} implies s_n + s_{n+1} = 1 + frac{s_{n+1}}{n+1}$.

(2) $s_n = nint_0^1 frac{x^{n-1}}{1+x} dx < nint_0^1 x^{n-1} dx = 1$

(3) $s_{n+1} - s_n = int_0^1 frac{d (x^{n+1}-x^n)}{1+x} = int_0^1 x^n frac{1-x}{(1+x)^2} dx > 0$



(2+3) $implies s = lim_{ntoinfty} s_n$ exists and (1+2) $implies s+s = 1 + 0 implies s = frac{1}{2}$.



In any event, $s_n$ can be evaluated exactly to $n (psi(n) - psi(frac{n}{2}) - ln{2})$ where $psi(x)$ is the diagamma function. Since $psi(x) approx ln(x) - frac{1}{2x} - frac{1}{12x^2} + frac{1}{120x^4} + ... $ as $x to infty$, we know:
$$s_n approx frac{1}{2} + frac{1}{4 n} - frac{1}{8 n^3} + ...$$






share|cite|improve this answer









$endgroup$





















    3












    $begingroup$

    Using the substitution $xmapsto x^{1/n}$ and Dominated Convergence,
    $$
    begin{align}
    lim_{ntoinfty}int_0^1frac{nx^{n-1}}{1+x},mathrm{d}x
    &=lim_{ntoinfty}int_0^1frac1{1+x^{1/n}},mathrm{d}x\
    &=int_0^1frac12,mathrm{d}x\
    &=frac12
    end{align}
    $$






    share|cite|improve this answer









    $endgroup$













    • $begingroup$
      Nice Robjohn . I have a doubt . When $displaystyle lim_{nrightarrow infty} $ and $0<x<1.$. Then $(x)^n=0.$ So $displaystyle lim_{nrightarrow infty}int^{1}_{0}frac{n x^{n-1}}{1+x}dx=0$. please explain me where i am wrong. Thanks
      $endgroup$
      – D Tiwari
      Oct 2 '18 at 7:16






    • 1




      $begingroup$
      @DurgeshTiwari: You cannot simply exchange limits and integrals. There are rules for doing so. Above, I used Dominated Convergence, which says that if $|f_n(x)|le g(x)$ for all $n$ and $xin[0,1]$ and $int_0^1g(x),mathrm{d}xltinfty$, then $$limlimits_{ntoinfty}int_0^1f_n(x),mathrm{d}x=int_0^1limlimits_{ntoinfty}f_n(x),mathrm{d}x$$ There is no such $g(x)$ that dominates $frac{nx^{n-1}}{1+x}$ for all $n$.
      $endgroup$
      – robjohn
      Oct 2 '18 at 8:34













    Your Answer





    StackExchange.ifUsing("editor", function () {
    return StackExchange.using("mathjaxEditing", function () {
    StackExchange.MarkdownEditor.creationCallbacks.add(function (editor, postfix) {
    StackExchange.mathjaxEditing.prepareWmdForMathJax(editor, postfix, [["$", "$"], ["\\(","\\)"]]);
    });
    });
    }, "mathjax-editing");

    StackExchange.ready(function() {
    var channelOptions = {
    tags: "".split(" "),
    id: "69"
    };
    initTagRenderer("".split(" "), "".split(" "), channelOptions);

    StackExchange.using("externalEditor", function() {
    // Have to fire editor after snippets, if snippets enabled
    if (StackExchange.settings.snippets.snippetsEnabled) {
    StackExchange.using("snippets", function() {
    createEditor();
    });
    }
    else {
    createEditor();
    }
    });

    function createEditor() {
    StackExchange.prepareEditor({
    heartbeatType: 'answer',
    autoActivateHeartbeat: false,
    convertImagesToLinks: true,
    noModals: true,
    showLowRepImageUploadWarning: true,
    reputationToPostImages: 10,
    bindNavPrevention: true,
    postfix: "",
    imageUploader: {
    brandingHtml: "Powered by u003ca class="icon-imgur-white" href="https://imgur.com/"u003eu003c/au003e",
    contentPolicyHtml: "User contributions licensed under u003ca href="https://creativecommons.org/licenses/by-sa/3.0/"u003ecc by-sa 3.0 with attribution requiredu003c/au003e u003ca href="https://stackoverflow.com/legal/content-policy"u003e(content policy)u003c/au003e",
    allowUrls: true
    },
    noCode: true, onDemand: true,
    discardSelector: ".discard-answer"
    ,immediatelyShowMarkdownHelp:true
    });


    }
    });














    draft saved

    draft discarded


















    StackExchange.ready(
    function () {
    StackExchange.openid.initPostLogin('.new-post-login', 'https%3a%2f%2fmath.stackexchange.com%2fquestions%2f292251%2flimit-of-s-n-int-limits-01-fracnxn-11x-dx-as-n-to-infty%23new-answer', 'question_page');
    }
    );

    Post as a guest















    Required, but never shown
























    5 Answers
    5






    active

    oldest

    votes








    5 Answers
    5






    active

    oldest

    votes









    active

    oldest

    votes






    active

    oldest

    votes









    12












    $begingroup$

    We simplify the formulate for $s_n$ by integrating by parts.



    begin{align}
    s_n &= intlimits_0^1 frac{nx^{n-1}}{1+x} d x \
    &= left[
    frac{1}{1+x} int nx^{n-1} d x
    - int frac{1}{left(1+xright)^2} left(int nx^{n-1} d xright) d x
    right]^1_0 \
    &= left[frac{1}{1+x} int nx^{n-1} d xright]^1_0
    - left[int frac{1}{left(1+xright)^2} left(int nx^{n-1} dxright) d xright]^1_0 \
    &= left[frac{x^n}{1+x}right]^1_0
    - left[int frac{x^n}{left(1+xright)^2} d xright]^1_0 \
    &= frac{1}{2} - intlimits_0^1 frac{x^n}{left(1+xright)^2} d x \
    end{align}



    Now we estimate the remaining integral in the expression
    begin{align}
    I(n) &= intlimits_0^1 frac{x^n}{left(1+x right)^2} d x \
    &leq intlimits_0^1 x^n d x \
    &= frac{1}{n+1}
    end{align}



    Hence, $I(n) to 0$ as $n to infty$.



    And so, the expression can be rewritten as
    begin{align}
    limlimits_{n to infty} s_n = frac{1}{2}
    end{align}






    share|cite|improve this answer











    $endgroup$













    • $begingroup$
      What do you mean by $I(n) = e^{log(x)n}$ in the last line?
      $endgroup$
      – Antonio Vargas
      Feb 1 '13 at 18:15










    • $begingroup$
      Added the explanation. I hope its correct.
      $endgroup$
      – user14082
      Feb 1 '13 at 18:23










    • $begingroup$
      It is not; you can't just pull a $log x$ out of the integral since you're integrating with respect to $x$.
      $endgroup$
      – Antonio Vargas
      Feb 1 '13 at 18:29










    • $begingroup$
      Arrrrgh. Thanks.
      $endgroup$
      – user14082
      Feb 1 '13 at 18:30






    • 4




      $begingroup$
      $I(n)leq int_0^1 x^n dx={1over n+1}.$
      $endgroup$
      – user940
      Feb 1 '13 at 19:19


















    12












    $begingroup$

    We simplify the formulate for $s_n$ by integrating by parts.



    begin{align}
    s_n &= intlimits_0^1 frac{nx^{n-1}}{1+x} d x \
    &= left[
    frac{1}{1+x} int nx^{n-1} d x
    - int frac{1}{left(1+xright)^2} left(int nx^{n-1} d xright) d x
    right]^1_0 \
    &= left[frac{1}{1+x} int nx^{n-1} d xright]^1_0
    - left[int frac{1}{left(1+xright)^2} left(int nx^{n-1} dxright) d xright]^1_0 \
    &= left[frac{x^n}{1+x}right]^1_0
    - left[int frac{x^n}{left(1+xright)^2} d xright]^1_0 \
    &= frac{1}{2} - intlimits_0^1 frac{x^n}{left(1+xright)^2} d x \
    end{align}



    Now we estimate the remaining integral in the expression
    begin{align}
    I(n) &= intlimits_0^1 frac{x^n}{left(1+x right)^2} d x \
    &leq intlimits_0^1 x^n d x \
    &= frac{1}{n+1}
    end{align}



    Hence, $I(n) to 0$ as $n to infty$.



    And so, the expression can be rewritten as
    begin{align}
    limlimits_{n to infty} s_n = frac{1}{2}
    end{align}






    share|cite|improve this answer











    $endgroup$













    • $begingroup$
      What do you mean by $I(n) = e^{log(x)n}$ in the last line?
      $endgroup$
      – Antonio Vargas
      Feb 1 '13 at 18:15










    • $begingroup$
      Added the explanation. I hope its correct.
      $endgroup$
      – user14082
      Feb 1 '13 at 18:23










    • $begingroup$
      It is not; you can't just pull a $log x$ out of the integral since you're integrating with respect to $x$.
      $endgroup$
      – Antonio Vargas
      Feb 1 '13 at 18:29










    • $begingroup$
      Arrrrgh. Thanks.
      $endgroup$
      – user14082
      Feb 1 '13 at 18:30






    • 4




      $begingroup$
      $I(n)leq int_0^1 x^n dx={1over n+1}.$
      $endgroup$
      – user940
      Feb 1 '13 at 19:19
















    12












    12








    12





    $begingroup$

    We simplify the formulate for $s_n$ by integrating by parts.



    begin{align}
    s_n &= intlimits_0^1 frac{nx^{n-1}}{1+x} d x \
    &= left[
    frac{1}{1+x} int nx^{n-1} d x
    - int frac{1}{left(1+xright)^2} left(int nx^{n-1} d xright) d x
    right]^1_0 \
    &= left[frac{1}{1+x} int nx^{n-1} d xright]^1_0
    - left[int frac{1}{left(1+xright)^2} left(int nx^{n-1} dxright) d xright]^1_0 \
    &= left[frac{x^n}{1+x}right]^1_0
    - left[int frac{x^n}{left(1+xright)^2} d xright]^1_0 \
    &= frac{1}{2} - intlimits_0^1 frac{x^n}{left(1+xright)^2} d x \
    end{align}



    Now we estimate the remaining integral in the expression
    begin{align}
    I(n) &= intlimits_0^1 frac{x^n}{left(1+x right)^2} d x \
    &leq intlimits_0^1 x^n d x \
    &= frac{1}{n+1}
    end{align}



    Hence, $I(n) to 0$ as $n to infty$.



    And so, the expression can be rewritten as
    begin{align}
    limlimits_{n to infty} s_n = frac{1}{2}
    end{align}






    share|cite|improve this answer











    $endgroup$



    We simplify the formulate for $s_n$ by integrating by parts.



    begin{align}
    s_n &= intlimits_0^1 frac{nx^{n-1}}{1+x} d x \
    &= left[
    frac{1}{1+x} int nx^{n-1} d x
    - int frac{1}{left(1+xright)^2} left(int nx^{n-1} d xright) d x
    right]^1_0 \
    &= left[frac{1}{1+x} int nx^{n-1} d xright]^1_0
    - left[int frac{1}{left(1+xright)^2} left(int nx^{n-1} dxright) d xright]^1_0 \
    &= left[frac{x^n}{1+x}right]^1_0
    - left[int frac{x^n}{left(1+xright)^2} d xright]^1_0 \
    &= frac{1}{2} - intlimits_0^1 frac{x^n}{left(1+xright)^2} d x \
    end{align}



    Now we estimate the remaining integral in the expression
    begin{align}
    I(n) &= intlimits_0^1 frac{x^n}{left(1+x right)^2} d x \
    &leq intlimits_0^1 x^n d x \
    &= frac{1}{n+1}
    end{align}



    Hence, $I(n) to 0$ as $n to infty$.



    And so, the expression can be rewritten as
    begin{align}
    limlimits_{n to infty} s_n = frac{1}{2}
    end{align}







    share|cite|improve this answer














    share|cite|improve this answer



    share|cite|improve this answer








    edited Feb 1 '13 at 19:33

























    answered Feb 1 '13 at 18:06







    user14082



















    • $begingroup$
      What do you mean by $I(n) = e^{log(x)n}$ in the last line?
      $endgroup$
      – Antonio Vargas
      Feb 1 '13 at 18:15










    • $begingroup$
      Added the explanation. I hope its correct.
      $endgroup$
      – user14082
      Feb 1 '13 at 18:23










    • $begingroup$
      It is not; you can't just pull a $log x$ out of the integral since you're integrating with respect to $x$.
      $endgroup$
      – Antonio Vargas
      Feb 1 '13 at 18:29










    • $begingroup$
      Arrrrgh. Thanks.
      $endgroup$
      – user14082
      Feb 1 '13 at 18:30






    • 4




      $begingroup$
      $I(n)leq int_0^1 x^n dx={1over n+1}.$
      $endgroup$
      – user940
      Feb 1 '13 at 19:19




















    • $begingroup$
      What do you mean by $I(n) = e^{log(x)n}$ in the last line?
      $endgroup$
      – Antonio Vargas
      Feb 1 '13 at 18:15










    • $begingroup$
      Added the explanation. I hope its correct.
      $endgroup$
      – user14082
      Feb 1 '13 at 18:23










    • $begingroup$
      It is not; you can't just pull a $log x$ out of the integral since you're integrating with respect to $x$.
      $endgroup$
      – Antonio Vargas
      Feb 1 '13 at 18:29










    • $begingroup$
      Arrrrgh. Thanks.
      $endgroup$
      – user14082
      Feb 1 '13 at 18:30






    • 4




      $begingroup$
      $I(n)leq int_0^1 x^n dx={1over n+1}.$
      $endgroup$
      – user940
      Feb 1 '13 at 19:19


















    $begingroup$
    What do you mean by $I(n) = e^{log(x)n}$ in the last line?
    $endgroup$
    – Antonio Vargas
    Feb 1 '13 at 18:15




    $begingroup$
    What do you mean by $I(n) = e^{log(x)n}$ in the last line?
    $endgroup$
    – Antonio Vargas
    Feb 1 '13 at 18:15












    $begingroup$
    Added the explanation. I hope its correct.
    $endgroup$
    – user14082
    Feb 1 '13 at 18:23




    $begingroup$
    Added the explanation. I hope its correct.
    $endgroup$
    – user14082
    Feb 1 '13 at 18:23












    $begingroup$
    It is not; you can't just pull a $log x$ out of the integral since you're integrating with respect to $x$.
    $endgroup$
    – Antonio Vargas
    Feb 1 '13 at 18:29




    $begingroup$
    It is not; you can't just pull a $log x$ out of the integral since you're integrating with respect to $x$.
    $endgroup$
    – Antonio Vargas
    Feb 1 '13 at 18:29












    $begingroup$
    Arrrrgh. Thanks.
    $endgroup$
    – user14082
    Feb 1 '13 at 18:30




    $begingroup$
    Arrrrgh. Thanks.
    $endgroup$
    – user14082
    Feb 1 '13 at 18:30




    4




    4




    $begingroup$
    $I(n)leq int_0^1 x^n dx={1over n+1}.$
    $endgroup$
    – user940
    Feb 1 '13 at 19:19






    $begingroup$
    $I(n)leq int_0^1 x^n dx={1over n+1}.$
    $endgroup$
    – user940
    Feb 1 '13 at 19:19













    9












    $begingroup$

    We use a basic result in calculus, namely $lim_{nto infty}nint_0^1x^nf(x) dx=f(1)$, $f$ continuous on $[0,1]$
    $$lim_{nto infty}left(frac{n}{n-1}times (n-1)intlimits_0^1 x^{n-1} frac{1}{(1+x)} dxright)=frac{1}{2}$$



    Chris.






    share|cite|improve this answer











    $endgroup$













    • $begingroup$
      What is the name or proof of this basic result?
      $endgroup$
      – Alex
      Feb 1 '13 at 19:53






    • 2




      $begingroup$
      @Alex: I don't know if it has a name. Our teacher told us once that the result may be easily proved with the calculus knowledge in high school. This link will help you: math.stackexchange.com/questions/168163/…
      $endgroup$
      – user 1357113
      Feb 1 '13 at 20:01






    • 1




      $begingroup$
      Nice solution Chris'ssister! :-) Your question and the corresponding answers provide a decent tool. :-)
      $endgroup$
      – user14082
      Feb 1 '13 at 20:32












    • $begingroup$
      @OrangeHarvester: Thanks! :-) are you Jayesh? (sister here now)
      $endgroup$
      – user 1357113
      Feb 1 '13 at 20:44












    • $begingroup$
      Yes. I am Jayesh. Name changed for a month.
      $endgroup$
      – user14082
      Feb 1 '13 at 22:15
















    9












    $begingroup$

    We use a basic result in calculus, namely $lim_{nto infty}nint_0^1x^nf(x) dx=f(1)$, $f$ continuous on $[0,1]$
    $$lim_{nto infty}left(frac{n}{n-1}times (n-1)intlimits_0^1 x^{n-1} frac{1}{(1+x)} dxright)=frac{1}{2}$$



    Chris.






    share|cite|improve this answer











    $endgroup$













    • $begingroup$
      What is the name or proof of this basic result?
      $endgroup$
      – Alex
      Feb 1 '13 at 19:53






    • 2




      $begingroup$
      @Alex: I don't know if it has a name. Our teacher told us once that the result may be easily proved with the calculus knowledge in high school. This link will help you: math.stackexchange.com/questions/168163/…
      $endgroup$
      – user 1357113
      Feb 1 '13 at 20:01






    • 1




      $begingroup$
      Nice solution Chris'ssister! :-) Your question and the corresponding answers provide a decent tool. :-)
      $endgroup$
      – user14082
      Feb 1 '13 at 20:32












    • $begingroup$
      @OrangeHarvester: Thanks! :-) are you Jayesh? (sister here now)
      $endgroup$
      – user 1357113
      Feb 1 '13 at 20:44












    • $begingroup$
      Yes. I am Jayesh. Name changed for a month.
      $endgroup$
      – user14082
      Feb 1 '13 at 22:15














    9












    9








    9





    $begingroup$

    We use a basic result in calculus, namely $lim_{nto infty}nint_0^1x^nf(x) dx=f(1)$, $f$ continuous on $[0,1]$
    $$lim_{nto infty}left(frac{n}{n-1}times (n-1)intlimits_0^1 x^{n-1} frac{1}{(1+x)} dxright)=frac{1}{2}$$



    Chris.






    share|cite|improve this answer











    $endgroup$



    We use a basic result in calculus, namely $lim_{nto infty}nint_0^1x^nf(x) dx=f(1)$, $f$ continuous on $[0,1]$
    $$lim_{nto infty}left(frac{n}{n-1}times (n-1)intlimits_0^1 x^{n-1} frac{1}{(1+x)} dxright)=frac{1}{2}$$



    Chris.







    share|cite|improve this answer














    share|cite|improve this answer



    share|cite|improve this answer








    edited Apr 13 '17 at 12:21









    Community

    1




    1










    answered Feb 1 '13 at 19:36









    user 1357113user 1357113

    22.4k877226




    22.4k877226












    • $begingroup$
      What is the name or proof of this basic result?
      $endgroup$
      – Alex
      Feb 1 '13 at 19:53






    • 2




      $begingroup$
      @Alex: I don't know if it has a name. Our teacher told us once that the result may be easily proved with the calculus knowledge in high school. This link will help you: math.stackexchange.com/questions/168163/…
      $endgroup$
      – user 1357113
      Feb 1 '13 at 20:01






    • 1




      $begingroup$
      Nice solution Chris'ssister! :-) Your question and the corresponding answers provide a decent tool. :-)
      $endgroup$
      – user14082
      Feb 1 '13 at 20:32












    • $begingroup$
      @OrangeHarvester: Thanks! :-) are you Jayesh? (sister here now)
      $endgroup$
      – user 1357113
      Feb 1 '13 at 20:44












    • $begingroup$
      Yes. I am Jayesh. Name changed for a month.
      $endgroup$
      – user14082
      Feb 1 '13 at 22:15


















    • $begingroup$
      What is the name or proof of this basic result?
      $endgroup$
      – Alex
      Feb 1 '13 at 19:53






    • 2




      $begingroup$
      @Alex: I don't know if it has a name. Our teacher told us once that the result may be easily proved with the calculus knowledge in high school. This link will help you: math.stackexchange.com/questions/168163/…
      $endgroup$
      – user 1357113
      Feb 1 '13 at 20:01






    • 1




      $begingroup$
      Nice solution Chris'ssister! :-) Your question and the corresponding answers provide a decent tool. :-)
      $endgroup$
      – user14082
      Feb 1 '13 at 20:32












    • $begingroup$
      @OrangeHarvester: Thanks! :-) are you Jayesh? (sister here now)
      $endgroup$
      – user 1357113
      Feb 1 '13 at 20:44












    • $begingroup$
      Yes. I am Jayesh. Name changed for a month.
      $endgroup$
      – user14082
      Feb 1 '13 at 22:15
















    $begingroup$
    What is the name or proof of this basic result?
    $endgroup$
    – Alex
    Feb 1 '13 at 19:53




    $begingroup$
    What is the name or proof of this basic result?
    $endgroup$
    – Alex
    Feb 1 '13 at 19:53




    2




    2




    $begingroup$
    @Alex: I don't know if it has a name. Our teacher told us once that the result may be easily proved with the calculus knowledge in high school. This link will help you: math.stackexchange.com/questions/168163/…
    $endgroup$
    – user 1357113
    Feb 1 '13 at 20:01




    $begingroup$
    @Alex: I don't know if it has a name. Our teacher told us once that the result may be easily proved with the calculus knowledge in high school. This link will help you: math.stackexchange.com/questions/168163/…
    $endgroup$
    – user 1357113
    Feb 1 '13 at 20:01




    1




    1




    $begingroup$
    Nice solution Chris'ssister! :-) Your question and the corresponding answers provide a decent tool. :-)
    $endgroup$
    – user14082
    Feb 1 '13 at 20:32






    $begingroup$
    Nice solution Chris'ssister! :-) Your question and the corresponding answers provide a decent tool. :-)
    $endgroup$
    – user14082
    Feb 1 '13 at 20:32














    $begingroup$
    @OrangeHarvester: Thanks! :-) are you Jayesh? (sister here now)
    $endgroup$
    – user 1357113
    Feb 1 '13 at 20:44






    $begingroup$
    @OrangeHarvester: Thanks! :-) are you Jayesh? (sister here now)
    $endgroup$
    – user 1357113
    Feb 1 '13 at 20:44














    $begingroup$
    Yes. I am Jayesh. Name changed for a month.
    $endgroup$
    – user14082
    Feb 1 '13 at 22:15




    $begingroup$
    Yes. I am Jayesh. Name changed for a month.
    $endgroup$
    – user14082
    Feb 1 '13 at 22:15











    5












    $begingroup$

    Here's a solution based on order statistics, similar to my answer here.



    Let $X_1,dots, X_n$ be i.i.d. uniform(0,1) random variables.
    The distribution function of $X$ is $F(x)=x$ and density $f(x)=1$ for $0leq xleq 1$.
    Now let $M=max(X_1,dots, X_n)$; its density function is
    $$f_M(x)=n F(x)^{n-1}f_X(x)=n,x^{n-1}text{ for }0leq xleq 1.$$
    Also, it is not hard to see that $Mto 1$ in distribution as $ntoinfty$.
    Now $$int_0^1 {n x^{n-1}over 1+x} ,dx =int_0^1 {1over 1+x}, f_M(x) ,dx
    =mathbb{E}left({1over 1+M}right).$$



    This converges to
    ${1over 1+1}={1over 2}$ as $ntoinfty$.






    share|cite|improve this answer











    $endgroup$













    • $begingroup$
      Thanks. A different solution. :-)
      $endgroup$
      – user14082
      Feb 1 '13 at 18:37












    • $begingroup$
      @MikeSpivey Yeah, it's the way I think. Some of the other answers here are great, too.
      $endgroup$
      – user940
      Feb 1 '13 at 21:06










    • $begingroup$
      @Byron: If I may say so, I often enjoy your answers that use probability and statistics to answer questions that appear to be outside of those fields. They are close enough to the way I think that I can appreciate them, but I often wouldn't think of those arguments myself.
      $endgroup$
      – Mike Spivey
      Feb 1 '13 at 21:09










    • $begingroup$
      @MikeSpivey Thanks for the kind words. I learn a lot from your answers, too.
      $endgroup$
      – user940
      Feb 1 '13 at 21:12
















    5












    $begingroup$

    Here's a solution based on order statistics, similar to my answer here.



    Let $X_1,dots, X_n$ be i.i.d. uniform(0,1) random variables.
    The distribution function of $X$ is $F(x)=x$ and density $f(x)=1$ for $0leq xleq 1$.
    Now let $M=max(X_1,dots, X_n)$; its density function is
    $$f_M(x)=n F(x)^{n-1}f_X(x)=n,x^{n-1}text{ for }0leq xleq 1.$$
    Also, it is not hard to see that $Mto 1$ in distribution as $ntoinfty$.
    Now $$int_0^1 {n x^{n-1}over 1+x} ,dx =int_0^1 {1over 1+x}, f_M(x) ,dx
    =mathbb{E}left({1over 1+M}right).$$



    This converges to
    ${1over 1+1}={1over 2}$ as $ntoinfty$.






    share|cite|improve this answer











    $endgroup$













    • $begingroup$
      Thanks. A different solution. :-)
      $endgroup$
      – user14082
      Feb 1 '13 at 18:37












    • $begingroup$
      @MikeSpivey Yeah, it's the way I think. Some of the other answers here are great, too.
      $endgroup$
      – user940
      Feb 1 '13 at 21:06










    • $begingroup$
      @Byron: If I may say so, I often enjoy your answers that use probability and statistics to answer questions that appear to be outside of those fields. They are close enough to the way I think that I can appreciate them, but I often wouldn't think of those arguments myself.
      $endgroup$
      – Mike Spivey
      Feb 1 '13 at 21:09










    • $begingroup$
      @MikeSpivey Thanks for the kind words. I learn a lot from your answers, too.
      $endgroup$
      – user940
      Feb 1 '13 at 21:12














    5












    5








    5





    $begingroup$

    Here's a solution based on order statistics, similar to my answer here.



    Let $X_1,dots, X_n$ be i.i.d. uniform(0,1) random variables.
    The distribution function of $X$ is $F(x)=x$ and density $f(x)=1$ for $0leq xleq 1$.
    Now let $M=max(X_1,dots, X_n)$; its density function is
    $$f_M(x)=n F(x)^{n-1}f_X(x)=n,x^{n-1}text{ for }0leq xleq 1.$$
    Also, it is not hard to see that $Mto 1$ in distribution as $ntoinfty$.
    Now $$int_0^1 {n x^{n-1}over 1+x} ,dx =int_0^1 {1over 1+x}, f_M(x) ,dx
    =mathbb{E}left({1over 1+M}right).$$



    This converges to
    ${1over 1+1}={1over 2}$ as $ntoinfty$.






    share|cite|improve this answer











    $endgroup$



    Here's a solution based on order statistics, similar to my answer here.



    Let $X_1,dots, X_n$ be i.i.d. uniform(0,1) random variables.
    The distribution function of $X$ is $F(x)=x$ and density $f(x)=1$ for $0leq xleq 1$.
    Now let $M=max(X_1,dots, X_n)$; its density function is
    $$f_M(x)=n F(x)^{n-1}f_X(x)=n,x^{n-1}text{ for }0leq xleq 1.$$
    Also, it is not hard to see that $Mto 1$ in distribution as $ntoinfty$.
    Now $$int_0^1 {n x^{n-1}over 1+x} ,dx =int_0^1 {1over 1+x}, f_M(x) ,dx
    =mathbb{E}left({1over 1+M}right).$$



    This converges to
    ${1over 1+1}={1over 2}$ as $ntoinfty$.







    share|cite|improve this answer














    share|cite|improve this answer



    share|cite|improve this answer








    edited Apr 13 '17 at 12:20









    Community

    1




    1










    answered Feb 1 '13 at 18:30







    user940



















    • $begingroup$
      Thanks. A different solution. :-)
      $endgroup$
      – user14082
      Feb 1 '13 at 18:37












    • $begingroup$
      @MikeSpivey Yeah, it's the way I think. Some of the other answers here are great, too.
      $endgroup$
      – user940
      Feb 1 '13 at 21:06










    • $begingroup$
      @Byron: If I may say so, I often enjoy your answers that use probability and statistics to answer questions that appear to be outside of those fields. They are close enough to the way I think that I can appreciate them, but I often wouldn't think of those arguments myself.
      $endgroup$
      – Mike Spivey
      Feb 1 '13 at 21:09










    • $begingroup$
      @MikeSpivey Thanks for the kind words. I learn a lot from your answers, too.
      $endgroup$
      – user940
      Feb 1 '13 at 21:12


















    • $begingroup$
      Thanks. A different solution. :-)
      $endgroup$
      – user14082
      Feb 1 '13 at 18:37












    • $begingroup$
      @MikeSpivey Yeah, it's the way I think. Some of the other answers here are great, too.
      $endgroup$
      – user940
      Feb 1 '13 at 21:06










    • $begingroup$
      @Byron: If I may say so, I often enjoy your answers that use probability and statistics to answer questions that appear to be outside of those fields. They are close enough to the way I think that I can appreciate them, but I often wouldn't think of those arguments myself.
      $endgroup$
      – Mike Spivey
      Feb 1 '13 at 21:09










    • $begingroup$
      @MikeSpivey Thanks for the kind words. I learn a lot from your answers, too.
      $endgroup$
      – user940
      Feb 1 '13 at 21:12
















    $begingroup$
    Thanks. A different solution. :-)
    $endgroup$
    – user14082
    Feb 1 '13 at 18:37






    $begingroup$
    Thanks. A different solution. :-)
    $endgroup$
    – user14082
    Feb 1 '13 at 18:37














    $begingroup$
    @MikeSpivey Yeah, it's the way I think. Some of the other answers here are great, too.
    $endgroup$
    – user940
    Feb 1 '13 at 21:06




    $begingroup$
    @MikeSpivey Yeah, it's the way I think. Some of the other answers here are great, too.
    $endgroup$
    – user940
    Feb 1 '13 at 21:06












    $begingroup$
    @Byron: If I may say so, I often enjoy your answers that use probability and statistics to answer questions that appear to be outside of those fields. They are close enough to the way I think that I can appreciate them, but I often wouldn't think of those arguments myself.
    $endgroup$
    – Mike Spivey
    Feb 1 '13 at 21:09




    $begingroup$
    @Byron: If I may say so, I often enjoy your answers that use probability and statistics to answer questions that appear to be outside of those fields. They are close enough to the way I think that I can appreciate them, but I often wouldn't think of those arguments myself.
    $endgroup$
    – Mike Spivey
    Feb 1 '13 at 21:09












    $begingroup$
    @MikeSpivey Thanks for the kind words. I learn a lot from your answers, too.
    $endgroup$
    – user940
    Feb 1 '13 at 21:12




    $begingroup$
    @MikeSpivey Thanks for the kind words. I learn a lot from your answers, too.
    $endgroup$
    – user940
    Feb 1 '13 at 21:12











    5












    $begingroup$

    Notice

    (1) $frac{s_n}{n} + frac{s_{n+1}}{n+1} = int_0^1 x^{n-1} dx = frac{1}{n} implies s_n + s_{n+1} = 1 + frac{s_{n+1}}{n+1}$.

    (2) $s_n = nint_0^1 frac{x^{n-1}}{1+x} dx < nint_0^1 x^{n-1} dx = 1$

    (3) $s_{n+1} - s_n = int_0^1 frac{d (x^{n+1}-x^n)}{1+x} = int_0^1 x^n frac{1-x}{(1+x)^2} dx > 0$



    (2+3) $implies s = lim_{ntoinfty} s_n$ exists and (1+2) $implies s+s = 1 + 0 implies s = frac{1}{2}$.



    In any event, $s_n$ can be evaluated exactly to $n (psi(n) - psi(frac{n}{2}) - ln{2})$ where $psi(x)$ is the diagamma function. Since $psi(x) approx ln(x) - frac{1}{2x} - frac{1}{12x^2} + frac{1}{120x^4} + ... $ as $x to infty$, we know:
    $$s_n approx frac{1}{2} + frac{1}{4 n} - frac{1}{8 n^3} + ...$$






    share|cite|improve this answer









    $endgroup$


















      5












      $begingroup$

      Notice

      (1) $frac{s_n}{n} + frac{s_{n+1}}{n+1} = int_0^1 x^{n-1} dx = frac{1}{n} implies s_n + s_{n+1} = 1 + frac{s_{n+1}}{n+1}$.

      (2) $s_n = nint_0^1 frac{x^{n-1}}{1+x} dx < nint_0^1 x^{n-1} dx = 1$

      (3) $s_{n+1} - s_n = int_0^1 frac{d (x^{n+1}-x^n)}{1+x} = int_0^1 x^n frac{1-x}{(1+x)^2} dx > 0$



      (2+3) $implies s = lim_{ntoinfty} s_n$ exists and (1+2) $implies s+s = 1 + 0 implies s = frac{1}{2}$.



      In any event, $s_n$ can be evaluated exactly to $n (psi(n) - psi(frac{n}{2}) - ln{2})$ where $psi(x)$ is the diagamma function. Since $psi(x) approx ln(x) - frac{1}{2x} - frac{1}{12x^2} + frac{1}{120x^4} + ... $ as $x to infty$, we know:
      $$s_n approx frac{1}{2} + frac{1}{4 n} - frac{1}{8 n^3} + ...$$






      share|cite|improve this answer









      $endgroup$
















        5












        5








        5





        $begingroup$

        Notice

        (1) $frac{s_n}{n} + frac{s_{n+1}}{n+1} = int_0^1 x^{n-1} dx = frac{1}{n} implies s_n + s_{n+1} = 1 + frac{s_{n+1}}{n+1}$.

        (2) $s_n = nint_0^1 frac{x^{n-1}}{1+x} dx < nint_0^1 x^{n-1} dx = 1$

        (3) $s_{n+1} - s_n = int_0^1 frac{d (x^{n+1}-x^n)}{1+x} = int_0^1 x^n frac{1-x}{(1+x)^2} dx > 0$



        (2+3) $implies s = lim_{ntoinfty} s_n$ exists and (1+2) $implies s+s = 1 + 0 implies s = frac{1}{2}$.



        In any event, $s_n$ can be evaluated exactly to $n (psi(n) - psi(frac{n}{2}) - ln{2})$ where $psi(x)$ is the diagamma function. Since $psi(x) approx ln(x) - frac{1}{2x} - frac{1}{12x^2} + frac{1}{120x^4} + ... $ as $x to infty$, we know:
        $$s_n approx frac{1}{2} + frac{1}{4 n} - frac{1}{8 n^3} + ...$$






        share|cite|improve this answer









        $endgroup$



        Notice

        (1) $frac{s_n}{n} + frac{s_{n+1}}{n+1} = int_0^1 x^{n-1} dx = frac{1}{n} implies s_n + s_{n+1} = 1 + frac{s_{n+1}}{n+1}$.

        (2) $s_n = nint_0^1 frac{x^{n-1}}{1+x} dx < nint_0^1 x^{n-1} dx = 1$

        (3) $s_{n+1} - s_n = int_0^1 frac{d (x^{n+1}-x^n)}{1+x} = int_0^1 x^n frac{1-x}{(1+x)^2} dx > 0$



        (2+3) $implies s = lim_{ntoinfty} s_n$ exists and (1+2) $implies s+s = 1 + 0 implies s = frac{1}{2}$.



        In any event, $s_n$ can be evaluated exactly to $n (psi(n) - psi(frac{n}{2}) - ln{2})$ where $psi(x)$ is the diagamma function. Since $psi(x) approx ln(x) - frac{1}{2x} - frac{1}{12x^2} + frac{1}{120x^4} + ... $ as $x to infty$, we know:
        $$s_n approx frac{1}{2} + frac{1}{4 n} - frac{1}{8 n^3} + ...$$







        share|cite|improve this answer












        share|cite|improve this answer



        share|cite|improve this answer










        answered Feb 1 '13 at 20:56









        achille huiachille hui

        95.7k5131258




        95.7k5131258























            3












            $begingroup$

            Using the substitution $xmapsto x^{1/n}$ and Dominated Convergence,
            $$
            begin{align}
            lim_{ntoinfty}int_0^1frac{nx^{n-1}}{1+x},mathrm{d}x
            &=lim_{ntoinfty}int_0^1frac1{1+x^{1/n}},mathrm{d}x\
            &=int_0^1frac12,mathrm{d}x\
            &=frac12
            end{align}
            $$






            share|cite|improve this answer









            $endgroup$













            • $begingroup$
              Nice Robjohn . I have a doubt . When $displaystyle lim_{nrightarrow infty} $ and $0<x<1.$. Then $(x)^n=0.$ So $displaystyle lim_{nrightarrow infty}int^{1}_{0}frac{n x^{n-1}}{1+x}dx=0$. please explain me where i am wrong. Thanks
              $endgroup$
              – D Tiwari
              Oct 2 '18 at 7:16






            • 1




              $begingroup$
              @DurgeshTiwari: You cannot simply exchange limits and integrals. There are rules for doing so. Above, I used Dominated Convergence, which says that if $|f_n(x)|le g(x)$ for all $n$ and $xin[0,1]$ and $int_0^1g(x),mathrm{d}xltinfty$, then $$limlimits_{ntoinfty}int_0^1f_n(x),mathrm{d}x=int_0^1limlimits_{ntoinfty}f_n(x),mathrm{d}x$$ There is no such $g(x)$ that dominates $frac{nx^{n-1}}{1+x}$ for all $n$.
              $endgroup$
              – robjohn
              Oct 2 '18 at 8:34


















            3












            $begingroup$

            Using the substitution $xmapsto x^{1/n}$ and Dominated Convergence,
            $$
            begin{align}
            lim_{ntoinfty}int_0^1frac{nx^{n-1}}{1+x},mathrm{d}x
            &=lim_{ntoinfty}int_0^1frac1{1+x^{1/n}},mathrm{d}x\
            &=int_0^1frac12,mathrm{d}x\
            &=frac12
            end{align}
            $$






            share|cite|improve this answer









            $endgroup$













            • $begingroup$
              Nice Robjohn . I have a doubt . When $displaystyle lim_{nrightarrow infty} $ and $0<x<1.$. Then $(x)^n=0.$ So $displaystyle lim_{nrightarrow infty}int^{1}_{0}frac{n x^{n-1}}{1+x}dx=0$. please explain me where i am wrong. Thanks
              $endgroup$
              – D Tiwari
              Oct 2 '18 at 7:16






            • 1




              $begingroup$
              @DurgeshTiwari: You cannot simply exchange limits and integrals. There are rules for doing so. Above, I used Dominated Convergence, which says that if $|f_n(x)|le g(x)$ for all $n$ and $xin[0,1]$ and $int_0^1g(x),mathrm{d}xltinfty$, then $$limlimits_{ntoinfty}int_0^1f_n(x),mathrm{d}x=int_0^1limlimits_{ntoinfty}f_n(x),mathrm{d}x$$ There is no such $g(x)$ that dominates $frac{nx^{n-1}}{1+x}$ for all $n$.
              $endgroup$
              – robjohn
              Oct 2 '18 at 8:34
















            3












            3








            3





            $begingroup$

            Using the substitution $xmapsto x^{1/n}$ and Dominated Convergence,
            $$
            begin{align}
            lim_{ntoinfty}int_0^1frac{nx^{n-1}}{1+x},mathrm{d}x
            &=lim_{ntoinfty}int_0^1frac1{1+x^{1/n}},mathrm{d}x\
            &=int_0^1frac12,mathrm{d}x\
            &=frac12
            end{align}
            $$






            share|cite|improve this answer









            $endgroup$



            Using the substitution $xmapsto x^{1/n}$ and Dominated Convergence,
            $$
            begin{align}
            lim_{ntoinfty}int_0^1frac{nx^{n-1}}{1+x},mathrm{d}x
            &=lim_{ntoinfty}int_0^1frac1{1+x^{1/n}},mathrm{d}x\
            &=int_0^1frac12,mathrm{d}x\
            &=frac12
            end{align}
            $$







            share|cite|improve this answer












            share|cite|improve this answer



            share|cite|improve this answer










            answered Jan 10 '15 at 12:29









            robjohnrobjohn

            265k27304626




            265k27304626












            • $begingroup$
              Nice Robjohn . I have a doubt . When $displaystyle lim_{nrightarrow infty} $ and $0<x<1.$. Then $(x)^n=0.$ So $displaystyle lim_{nrightarrow infty}int^{1}_{0}frac{n x^{n-1}}{1+x}dx=0$. please explain me where i am wrong. Thanks
              $endgroup$
              – D Tiwari
              Oct 2 '18 at 7:16






            • 1




              $begingroup$
              @DurgeshTiwari: You cannot simply exchange limits and integrals. There are rules for doing so. Above, I used Dominated Convergence, which says that if $|f_n(x)|le g(x)$ for all $n$ and $xin[0,1]$ and $int_0^1g(x),mathrm{d}xltinfty$, then $$limlimits_{ntoinfty}int_0^1f_n(x),mathrm{d}x=int_0^1limlimits_{ntoinfty}f_n(x),mathrm{d}x$$ There is no such $g(x)$ that dominates $frac{nx^{n-1}}{1+x}$ for all $n$.
              $endgroup$
              – robjohn
              Oct 2 '18 at 8:34




















            • $begingroup$
              Nice Robjohn . I have a doubt . When $displaystyle lim_{nrightarrow infty} $ and $0<x<1.$. Then $(x)^n=0.$ So $displaystyle lim_{nrightarrow infty}int^{1}_{0}frac{n x^{n-1}}{1+x}dx=0$. please explain me where i am wrong. Thanks
              $endgroup$
              – D Tiwari
              Oct 2 '18 at 7:16






            • 1




              $begingroup$
              @DurgeshTiwari: You cannot simply exchange limits and integrals. There are rules for doing so. Above, I used Dominated Convergence, which says that if $|f_n(x)|le g(x)$ for all $n$ and $xin[0,1]$ and $int_0^1g(x),mathrm{d}xltinfty$, then $$limlimits_{ntoinfty}int_0^1f_n(x),mathrm{d}x=int_0^1limlimits_{ntoinfty}f_n(x),mathrm{d}x$$ There is no such $g(x)$ that dominates $frac{nx^{n-1}}{1+x}$ for all $n$.
              $endgroup$
              – robjohn
              Oct 2 '18 at 8:34


















            $begingroup$
            Nice Robjohn . I have a doubt . When $displaystyle lim_{nrightarrow infty} $ and $0<x<1.$. Then $(x)^n=0.$ So $displaystyle lim_{nrightarrow infty}int^{1}_{0}frac{n x^{n-1}}{1+x}dx=0$. please explain me where i am wrong. Thanks
            $endgroup$
            – D Tiwari
            Oct 2 '18 at 7:16




            $begingroup$
            Nice Robjohn . I have a doubt . When $displaystyle lim_{nrightarrow infty} $ and $0<x<1.$. Then $(x)^n=0.$ So $displaystyle lim_{nrightarrow infty}int^{1}_{0}frac{n x^{n-1}}{1+x}dx=0$. please explain me where i am wrong. Thanks
            $endgroup$
            – D Tiwari
            Oct 2 '18 at 7:16




            1




            1




            $begingroup$
            @DurgeshTiwari: You cannot simply exchange limits and integrals. There are rules for doing so. Above, I used Dominated Convergence, which says that if $|f_n(x)|le g(x)$ for all $n$ and $xin[0,1]$ and $int_0^1g(x),mathrm{d}xltinfty$, then $$limlimits_{ntoinfty}int_0^1f_n(x),mathrm{d}x=int_0^1limlimits_{ntoinfty}f_n(x),mathrm{d}x$$ There is no such $g(x)$ that dominates $frac{nx^{n-1}}{1+x}$ for all $n$.
            $endgroup$
            – robjohn
            Oct 2 '18 at 8:34






            $begingroup$
            @DurgeshTiwari: You cannot simply exchange limits and integrals. There are rules for doing so. Above, I used Dominated Convergence, which says that if $|f_n(x)|le g(x)$ for all $n$ and $xin[0,1]$ and $int_0^1g(x),mathrm{d}xltinfty$, then $$limlimits_{ntoinfty}int_0^1f_n(x),mathrm{d}x=int_0^1limlimits_{ntoinfty}f_n(x),mathrm{d}x$$ There is no such $g(x)$ that dominates $frac{nx^{n-1}}{1+x}$ for all $n$.
            $endgroup$
            – robjohn
            Oct 2 '18 at 8:34




















            draft saved

            draft discarded




















































            Thanks for contributing an answer to Mathematics Stack Exchange!


            • Please be sure to answer the question. Provide details and share your research!

            But avoid



            • Asking for help, clarification, or responding to other answers.

            • Making statements based on opinion; back them up with references or personal experience.


            Use MathJax to format equations. MathJax reference.


            To learn more, see our tips on writing great answers.




            draft saved


            draft discarded














            StackExchange.ready(
            function () {
            StackExchange.openid.initPostLogin('.new-post-login', 'https%3a%2f%2fmath.stackexchange.com%2fquestions%2f292251%2flimit-of-s-n-int-limits-01-fracnxn-11x-dx-as-n-to-infty%23new-answer', 'question_page');
            }
            );

            Post as a guest















            Required, but never shown





















































            Required, but never shown














            Required, but never shown












            Required, but never shown







            Required, but never shown

































            Required, but never shown














            Required, but never shown












            Required, but never shown







            Required, but never shown







            Popular posts from this blog

            Mario Kart Wii

            What does “Dominus providebit” mean?

            Antonio Litta Visconti Arese